- #1
mpapachristou
- 3
- 0
Today, I made the following question at Physics SE, but the community redirected me here
In short, I have made a proof for EF/MF phase difference near an oscilating dipole that "vanishes" far away from the dipole. So, what I am asking for is a proof verification.
The physics SE post:
http://physics.stackexchange.com/questions/166091/proof-verifying-on-em-waves-generated-by-an-oscilating-dipole?noredirect=1#comment350272_166091[/URL]
In short, I have made a proof for EF/MF phase difference near an oscilating dipole that "vanishes" far away from the dipole. So, what I am asking for is a proof verification.
The physics SE post:
http://physics.stackexchange.com/questions/166091/proof-verifying-on-em-waves-generated-by-an-oscilating-dipole?noredirect=1#comment350272_166091[/URL]
Last edited by a moderator: